Is this solution to a limit equation correct?

  • Thread starter Thread starter nedfin
  • Start date Start date
  • Tags Tags
    Limit
Click For Summary
The discussion centers on the limit equation and the confusion regarding its equivalence to expressions involving infinity. The initial limit, as (x,y) approaches (∞,0), is clarified to be 0, not 1, because (x-y)^x approaches infinity. The terms "inf^-inf" and "infinitesimal" are deemed invalid in this context. The conclusion emphasizes that the original answer of 1 is incorrect, and the limit indeed evaluates to 0. The conversation also encourages the use of LaTeX for clearer mathematical communication.
nedfin
Messages
2
Reaction score
1
Homework Statement
I ran into this equation online and an interested in why my answer might be wrong. What if anything is wrong with my logic.
Relevant Equations
1-(1/((x-y)^x))
lim(x,y) -> (inf,0)
Why is this not equivalent to

1 - inf^-inf,

Or 1 - infinitesimal ,

Or 1 ?

My answer was 1, which I told is incorrect.
 
Last edited:
Physics news on Phys.org
<br /> \lim _{(x,y)\to (\infty,0)} \frac{1}{(x-y)^{x}} = 0<br />
because ##(x-y)^x \to \infty ## as ##(x,y)\to (\infty,0)##. It is not equivalent to the first two items. ##\infty ^{-\infty}## is not a thing and neither is "infinitesimal". The initial limit is ##1## as you say.
 
nuuskur said:
<br /> \lim _{(x,y)\to (\infty,0)} \frac{1}{(x-y)^{x}} = 0<br />
because ##(x-y)^x \to \infty ## as ##(x,y)\to (\infty,0)##. It is not equivalent to the first two items. ##\infty ^{-\infty}## is not a thing and neither is "infinitesimal". The initial limit is ##1## as you say.
Thanks
 
nedfin said:
Homework Statement:: I ran into this equation online and an interested in why my answer might be wrong. What if anything is wrong with my logic.
Relevant Equations:: 1-(1/((x-y)^x))
lim(x,y) -> (inf,0)

Why is this not equivalent to

1 - inf^-inf,

Or 1 - infinitesimal ,

Or 1 ?

My answer was 1, which I told is incorrect.
Welcome to PF. :smile: I'm glad that @nuuskur was able to help you.

BTW, please consider learning LaTeX to post equations in discussion forums. You can find a "LaTeX Guide" link at the bottom of the Edit Window. It makes math equations *much* easier to read (as you can see by nuuskru's post). :smile:
 
Question: A clock's minute hand has length 4 and its hour hand has length 3. What is the distance between the tips at the moment when it is increasing most rapidly?(Putnam Exam Question) Answer: Making assumption that both the hands moves at constant angular velocities, the answer is ## \sqrt{7} .## But don't you think this assumption is somewhat doubtful and wrong?

Similar threads

  • · Replies 3 ·
Replies
3
Views
1K
  • · Replies 4 ·
Replies
4
Views
959
Replies
3
Views
8K
Replies
10
Views
1K
  • · Replies 4 ·
Replies
4
Views
2K
  • · Replies 11 ·
Replies
11
Views
2K
  • · Replies 8 ·
Replies
8
Views
1K
Replies
2
Views
1K
  • · Replies 1 ·
Replies
1
Views
1K
  • · Replies 3 ·
Replies
3
Views
2K